Đến nội dung

Gachdptrai12 nội dung

Có 274 mục bởi Gachdptrai12 (Tìm giới hạn từ 18-04-2020)



Sắp theo                Sắp xếp  

#639209 VMF's Marathon Bất Đẳng Thức Olympic

Đã gửi bởi Gachdptrai12 on 09-06-2016 - 20:27 trong Bất đẳng thức và cực trị

Bài toán 28. (Tập huấn đội tuyển Việt Nam 2016). Cho các số thực không âm $a,b,c$. Chứng minh rằng
\[ \sqrt{a^2+2bc}+\sqrt{b^2+2ca}+\sqrt{c^2+2ab} \leq \sqrt{a^2+b^2+c^2}+2\sqrt{ab+bc+ca}.\]

 Lời giải bài 28. Phân tích về dạng S.O.S ta được
$\sum \frac{(a-b)^{2}}{\sqrt{c^{2}+2ab}+\sqrt{a^{2}+b^{2}+c^{2}}}\geq \frac{\sum (a-b)^{2}}{\sqrt{ab+bc+ca}+\sqrt{a^{2}+b^{2}+c^{2}}}$
Áp dụng bất đẳng thức C-S ta có
$\sum \frac{(a-b)^{2}}{\sqrt{c^{2}+2ab}+\sqrt{a^{2}+b^{2}+c^{2}}}\geq \frac{[\sum (a-b)^{2}]^{2}}{(a-b)^{2}(\sqrt{c^{2}+2ab}+\sqrt{a^{2}+b^{2}+c^{2}})}$
Ta chỉ cần chứng minh
$(\sqrt{ab+bc+ca}+\sqrt{a^{2}+b^{2}+c^{2}})\sum (a-b)^{2}\geq \sum (a-b)^{2}(\sqrt{c^{2}+2ab}+\sqrt{a^{2}+b^{2}+c^{2}})$

$\Leftrightarrow\sqrt{ab+bc+ca}\sum (a-b)^{2} \geq \sum (a-b)^{2}\sqrt{c^{2}+2ab}$
$\Leftrightarrow \sum (a-b)^{2}(\sqrt{ab+bc+ca}-\sqrt{c^{2}+2ab})\geq 0$
Bây giờ ta sử dụng bất đẳng thức $\sqrt{x}-\sqrt{y}\geq \frac{x-y}{2\sqrt{x}}$ ta có
$\sum (a-b)^{2}(\sqrt{ab+bc+ca}-\sqrt{c^{2}+2ab})\geq \frac{\sum (a-b)^{2}[ab+bc+ca-2ab-c^{2}]}{2\sqrt{ab+bc+ca}}= \frac{\sum (a-b)^{2}(a-c)(b-c)}{\sqrt{ab+bc+ca}}$
Mà $\sum (a-b)^{2}(a-c)(b-c)=0$ nên ta có điều phải chứng minh

 

 Bài toán 29. (Võ Quốc Bá Cẩn) Cho $a,b,c$ là các số thực không âm chứng minh

$\sqrt{4a^{2}+bc}+\sqrt{4b^{2}+ca}+\sqrt{4c^{2}+ab}\leq \frac{5}{2}(a+b+c)$




#635456 VMF's Marathon Bất Đẳng Thức Olympic

Đã gửi bởi Gachdptrai12 on 25-05-2016 - 16:04 trong Bất đẳng thức và cực trị

Bài toán 8. Cho $a,b,c$ là các số thực dương. Chứng minh

\[\frac{\sqrt{a+b}}{c}+\frac{\sqrt{b+c}}{a}+\frac{\sqrt{a+c}}{b}\geq 3\sqrt{\frac{2(a+b+c)}{ab+bc+ca}}\]




#642372 VMF's Marathon Bất Đẳng Thức Olympic

Đã gửi bởi Gachdptrai12 on 26-06-2016 - 23:50 trong Bất đẳng thức và cực trị

Bài toán 37. (AoPS) . Cho các số thực không âm $a,b,c$. Chứng minh rằng
$$\sqrt{a^2+bc+b^2}+\sqrt{b^2+ca+c^2}+\sqrt{c^2+ab+a^2} \geq \sqrt{4(a^2+b^2+c^2)+5(ab+bc+ca)}.$$

1 lời giải khác giả sử $a=min(a,b,c)$ ta dễ dàng chứng minh được $\sqrt{a^{2}+bc+b^{2}}+\sqrt{b^{2}+ca+c^{2}}\geq \sqrt{a^{2}+ac+b^{2}}+\sqrt{b^{2}+bc+c^{2}}$
áp dụng $(x+y)^{2}=2(x^{2}+y^{2})-(x-y)^{2}$ ta có $(\sqrt{a^{2}+ac+b^{2}}+\sqrt{a^{2}+ab+c^{2}})^{2}=2(2a^{2}+2a(b+c)+b^{2}+c^{2})-\frac{(b-c)^{2}(b+c-a)^{2}}{(\sqrt{a^{2}+ac+b^{2}}+\sqrt{a^{2}+ab+c^{2}})^{2}}\geq 2(2a^{2}+2a(b+c)+b^{2}+c^{2})-(b-c)^{2}$$=4a^{2}+2a(b+c)+(b+c)^{2}$
ta sẽ chứng minh $\sqrt{4a^{2}+2a+(b+c)^{2}}+\sqrt{b^{2}+bc+c^{2}}\geq \sqrt{4(\sum a^{2})+5\sum ab}\Leftrightarrow 2\sqrt{(4a^{2}+2a(b+c)+(b+c)^{2})(b^{2}+bc+c^{2})}\geq 3a(b+c)+2(b^{2}+bc+c^{2}) \Leftrightarrow 4(4a^{2}+2a(b+c)+(b+c)^{2})(b^{2}+bc+c^{2})\geq 9a^{2}(b+c)^{2}+12a(b+c)(b^{2}+bc+c^{2})+4(b^{2}+bc+c^{2})^{2}\Leftrightarrow 3a^{2}(b-c)^{2}+4(a-b)(a-c)(b^{2}+bc+c^{2})\geq 0$ dpcm




#635882 VMF's Marathon Bất Đẳng Thức Olympic

Đã gửi bởi Gachdptrai12 on 27-05-2016 - 10:06 trong Bất đẳng thức và cực trị

Bài toán 12. (Phan Hồng Sơn) Cho $a,b,c>0$ thỏa $a+b+c=3$. Chứng minh rằng

$\frac{a^{2}}{\sqrt{bc+(a-1)^{2}}}+\frac{b^{2}}{\sqrt{ca+(b-1)^{2}}}+\frac{c^{2}}{\sqrt{ab+(c-1)^{2}}}\geq 3$




#641122 VMF's Marathon Bất Đẳng Thức Olympic

Đã gửi bởi Gachdptrai12 on 18-06-2016 - 20:17 trong Bất đẳng thức và cực trị

Bài 35/1: Giả sử $c=min(a,b,c)$.
Bất đẳng thức đã cho tương đương:
$\frac{7(a^2+b^2+c^2-ab-bc-ca)}{(a+b+c)(\sqrt{3(a^2+b^2+c^2)}+a+b+c)} \geq \frac{(a-b)^2(a+b)+(a-c)(b-c)(b+c)}{2(a^3+b^3+c^3)}$.
Do $(a-b)^2(a+b)+(a-c)(b-c)(b+c) \leq (a+b+c)(a^2+b^2+c^2-ab-bc-ca)$ nên ta chỉ cần chứng minh:
$14(a^3+b^3+c^3) \geq (a+b+c)^2\sqrt{3(a^2+b^2+c^2)}$.
Sử dụng Cauchy-Schwarz ta có:
$14(a^3+b^3+c^3) \geq 14\frac{(a^2+b^2+c^2)^2}{a+b+c} \geq \frac{14}{9}.\frac{(a+b+c)^3\sqrt{3(a^2+b^2+c^2)}}{a+b+c} \geq (a+b+c)^2\sqrt{3(a^2+b^2+c^2)} \blacksquare$.

$(a+b+c)^{3}$ mất đâu rồi bạn :3 




#635915 VMF's Marathon Bất Đẳng Thức Olympic

Đã gửi bởi Gachdptrai12 on 27-05-2016 - 12:38 trong Bất đẳng thức và cực trị

Bài toán 10. Cho $a, b, c >0$. Chứng minh rằng:
\[(a+b+c)\left(\dfrac{a}{b^2+c^2} + \dfrac{b}{c^2+a^2} + \dfrac{c}{a^2+b^2}\right) \geq 4+ \dfrac{4a^2b^2c^2}{(a^2+b^2)(b^2+c^2)(c^2+a^2)}\]

 Lời giải 3 cho bài 10:
 Viết bài toán dưới dạng $pqr$ là $p(11p^{2}-21q)r+(p^{2}-4q)(p^{2}-2q)(p^2+q)\geq0$
 Dễ thấy hệ số cuả $r$ là số dương nên theo phương pháp ABC thì cực trị đạt được tại 2 biến bằng nhau và một biến bằng $0$
 TH1: $b=c$ và do tính đồng bậc ta có thể giả sử $b=1$, do đó bất đẳng thức ban đầu trở thành
$(a+2)(\frac{a}{2}+\frac{2}{a^{2}+1}\geq 4+\frac{4a^2}{2(a^{2}+1)^{2}}$
$\Leftrightarrow a(a-1)^{2}(4a^{3}+4a^{2}+a+6)\geq0$ (hiển nhiên đúng)
 TH2: $c=0$ bất đẳng thức ban đầu trở thành
$(a+b)(\frac{a}{b^{2}}+\frac{b}{a^{2}})\geq4$
Tương đương $(a+b)(a^{3}+b^{3})\geq4a^{2}b^{2}$ (hiển nhiên đúng theo AM-GM)
 Vậy bất đẳng thức được chứng minh

 

PS




#638087 VMF's Marathon Bất Đẳng Thức Olympic

Đã gửi bởi Gachdptrai12 on 04-06-2016 - 21:18 trong Bất đẳng thức và cực trị

Bài toán 25. (Võ Quốc Bá Cẩn) Cho $a,b,c$ là các số thực không âm, chứng minh

$\dfrac{4(a^{2}+b^{2}+c^{2})}{ab+bc+ca}+\sqrt{3}\left(\frac{a+2b}{\sqrt{a^{2}+2b^{2}}}+\frac{b+2c}{\sqrt{b^{2}+2c^{2}}}+\frac{c+2a}{\sqrt{c^{2}+2a^{2}}}\right)\geq 13$




#635999 VMF's Marathon Bất Đẳng Thức Olympic

Đã gửi bởi Gachdptrai12 on 27-05-2016 - 17:24 trong Bất đẳng thức và cực trị

Biến đổi theo $p,q,r$ ta được $: \frac{3r-2q+1}{q-r} \geq \sqrt{\frac{4-27r}{4q}} $
Áp dụng $: r \geq max \left \{ 0, \frac{4q-1}{9} \right \} (1)$
TH1 $: q \leq 0,25 \Rightarrow VT (1) \geq \frac{-2q+1}{q} $
Cần cm $: \frac{-2q+1}{q} \geq \sqrt{\frac{1}{q}} (2) $
Vì $q>0$ nên $: (2) \Leftrightarrow (q-1)(4q-1) \geq 0 (đúng ) $
TH2 $: q \geq 0,25 $
p\s $:$ Ai rảnh xét hộ e TH $2 $

 Bạn ơi bạn chưa xét đượ̣c hàm đó là đồng biến hay nghịch biến theo $r$ đâu mà cho vô ầm ầm thế




#640745 VMF's Marathon Bất Đẳng Thức Olympic

Đã gửi bởi Gachdptrai12 on 16-06-2016 - 20:10 trong Bất đẳng thức và cực trị

Bài toán 33. (AoPS) Cho các số thực $a,b,c$ thỏa mãn $a,b,c\geq 1$ và $a+b+c+2=abc$. Chứng minh rằng

\[\dfrac{\sqrt{ab-1}+\sqrt{bc-1}+\sqrt{ca-1}}{\sqrt[4]{abc}}\leq \dfrac{1}{2}(\sqrt{a}+\sqrt{b}+\sqrt{c})\sqrt{\dfrac{1}{\sqrt{a}}+\dfrac{1}{\sqrt{b}}+\dfrac{1}{\sqrt{c}}}\]

Loi Giai:BĐT$\Leftrightarrow \sum \sqrt{ab-1}\leq \frac{1}{2}(\sqrt{a}+\sqrt{b}+\sqrt{c})(\sqrt{\sqrt{ab}+\sqrt{bc}+\sqrt{ca}})\Leftrightarrow 2\sum \frac{x(x+y+z)}{yz}\leq (\sqrt{a}+\sqrt{b}+\sqrt{c})(\sqrt{\sqrt{ab}+\sqrt{bc}+\sqrt{ca}})\Leftrightarrow 4\sum \frac{x(x+y+z)}{yz}+8 \sum \frac{x+y+z}{\sqrt{xy}}\leq (a+b+c+2(\sqrt{ab}+\sqrt{bc}+\sqrt{ca}))(\sqrt{ab}+\sqrt{bc}+\sqrt{ca})$

áp dụng bđt schur dạng phân thức $a+b+c\geq 2(\sqrt{ab}+\sqrt{bc}+\sqrt{ca})-\frac{9abc}{2(\sqrt{a}+\sqrt{b}+\sqrt{c})}$

ta cần chứng minh $4(\sqrt{ab}+\sqrt{bc}+\sqrt{ca})^{2}-\frac{9\sqrt{abc}(\sqrt{ab}+\sqrt{bc+\sqrt{ca}})}{\sqrt{a}+\sqrt{b}+\sqrt{c}}\geq 4\sum \frac{x(x+y+z)}{yz}+8\sum \frac{x+y+z}{\sqrt{xy}}$

mà$(\sqrt{ab}+\sqrt{bc}+\sqrt{ca})^{2}= \sum \frac{(x+y)(x+z)}{xy}+2\sum \frac{(y+z)\sqrt{(x+y)(x+z)}}{x\sqrt{yz}}\geq \sum \frac{(x+y)(x+z)}{xy}+2\sum \frac{(y+z)(x+\sqrt{yz})}{x\sqrt{yz}}= \sum \frac{(x+y)(x+z)}{xy}+2\sum \frac{y+z}{x}+2\sum \frac{x+y}{\sqrt{xy}}$

ta sẽ chứng minh $8\sum \frac{y+z}{x}-8\sum \frac{z}{\sqrt{xy}}+12\geq \frac{9\sqrt{abc}(\sqrt{ab}+\sqrt{bc+\sqrt{ca}})}{\sqrt{a}+\sqrt{b}+\sqrt{c}}$

mà $8\sum \frac{z}{\sqrt{xy}}\leq 4\sum (\frac{z}{x}+\frac{z}{y})= 4\sum \frac{x+y}{z}$

vậy ta có $4\sum \frac{x+y}{z}+12\geq \frac{9\sqrt{abc}(\sqrt{ab}+\sqrt{bc}+\sqrt{ca})}{\sqrt{a}+\sqrt{b}+\sqrt{c}}\Leftrightarrow 2(\sum \frac{1}{z})(\sum (x+y))\geq \frac{9\sqrt{abc}(\sqrt{ab}+\sqrt{bc}+\sqrt{ca})}{\sqrt{a}+\sqrt{b}+\sqrt{c}}$ 

áp dụng C-S $2(\sum (x+y))(\sum \frac{1}{z} )\geq 2(\sum \sqrt{\frac{y+z}{x}})^{2}= 2(\sqrt{a}+\sqrt{b}+\sqrt{c})^{2}$

ta cần chứng minh $2(\sqrt{a}+\sqrt{b}+\sqrt{c})^{3}\geq 9\sqrt{abc}(\sum \sqrt{ab})=9(\sum \sqrt{ab})\sqrt{a+b+c+2}$ mà $a+b+c\geq6$ nên ta có $\sqrt{a+b+c+2}\leq \frac{2}{\sqrt{3}}\sqrt{a+b+c}$$(\sqrt{a}+\sqrt{b}+\sqrt{c})^{3}\geq$

ta chứng minh  $3\sqrt{3(a+b+c)(\sum \sqrt{ab})^{2}}$ mà theo AM-GM $27(a+b+c)(\sqrt{ab}+\sqrt{bc}+\sqrt{ca})\leq (\sqrt{a}+\sqrt{b}+\sqrt{c})^{6}$ vậy ta có đpcm

bài 35:VQBC cho $a,b,c$$2(\frac{a}b{+\frac{b}c{+\frac{c}{a}}})+1\geq \frac{21(a^{2}+b^{2}+c^{2})}{(a+b+c)^{2}}$ là các số thực ko âm chứng minh

P/s cho mình được phá lệ nha vì bài 35 mình thấy hay nên cho mình đăng 1 bài dùng 35 làm bổ đề và bài 35 cũng rất thú vị cho những ai giải các bđt hoán vị. Thím Long cho cái P/S zô hide hộ :3

Bài 35/1:(Sưu Tầm) cho $a,b,c >0$ chứng minh$\frac{7\sqrt{3(a^{2}+b^{2}+c^{2})}}{a+b+c}+\frac{a^{2}b+b^{2}c+c^{2}a}{a^{3}+b^{3}+c^{3}}\geq 8$




#636292 VMF's Marathon Bất Đẳng Thức Olympic

Đã gửi bởi Gachdptrai12 on 28-05-2016 - 17:04 trong Bất đẳng thức và cực trị

Bài 13. (Sưu tầm) Cho $a,b,c>0$ thỏa $a+b+c=1$. Chứng minh

\[\frac{a}{b+c}+\frac{b}{a+c}+\frac{c}{a+b} \geq \sqrt{\frac{4-27abc}{4(ab+bc+ac)}}\]

Lời giải bài 13: Không mất tính tổng quát giả sử $a\geq b\geq c$, ta có một đẳng thức$(x+y+z)^{2}=3(xy+yz+zx)+\frac{1}{2}\sum (x-y)^{2}$

Từ đó ta có

BĐT$\Leftrightarrow (\frac{a}{b+c}+\frac{b}{c+a}+\frac{c}{a+b})^{2}\geq \frac{4(a+b+c)^{3}-27abc}{4(a+b+c)(ab+bc+ca)}$

Áp dụng đẳng thức trên ta có

$(\sum \frac{a}{b+c})^{2}=3\sum \frac{ab}{(a+c)(b+c)}+\frac{1}{2}\sum (\frac{a}{b+c}-\frac{b}{c+a})^{2}$

$=3\frac{\sum ab(a+b)}{(a+b)(b+c)(c+a)}+\frac{(a+b+c)^{2}}{2}\sum \frac{(a-b)^{2}}{(a+c)^{2}(b+c)^{2}}$

Mà $3 \frac{\sum ab(a+b)}{(a+b)(b+c)(c+a)}-\frac{9}{4}=\frac{3}{4}\sum \frac{c(a-b)^{2}}{(a+b)(b+c)(c+a)}$

Và $\frac{4(a+b+c)^{3}-27abc}{4(a+b+c)(ab+bc+ca)}-\frac{9}{4}=\frac{4(a^{3}+b^{3}+c^{3}-3abc)+3(a+b)(b+c)(c+a)-24abc}{4(a+b+c)(ab+bc+ca)}$

$= \sum \frac{(a-b)^{2}}{2(ab+bc+ca)}+\frac{3}{4}\sum \frac{c(a-b)^{2}}{(a+b+c)(ab+bc+ca)}\leq \sum \frac{(a-b)^{2}}{2(ab+bc+ca)}+\frac{3}{4}\sum \frac{c(a-b)^{2}}{(a+b)(b+c)(c+a)}$

Từ đó ta chỉ cần chứng minh được

$\sum (a-b)^{2}[\frac{(a+b+c)^{2}}{2(a+c)^{2}(b+c)^{2}}-\frac{1}{2(ab+bc+ca)}]\Leftrightarrow \sum S_{c}(a-b)^{2}\geq 0$

Dễ thấy $S_{c}\geq S_{b}\geq S_{a}$ ta có

$S_{a}+S_{b}=\frac{(a+b+c)^{2}}{2(a+b)^{2}(b+c^{2})}+\frac{(a+b+c)^{2}}{2(a+c)^{2}(b+c^{2})}-\frac{1}{ab+bc+ca}=\frac{(a+b+c)^{2}}{(a+b)^{2}(a+c)(b+c)}-\frac{1}{ab+bc+ca}\geq \frac{a+b+c}{(a+b)(b+c)(c+a)}-\frac{1}{ab+bc+ca}=\frac{abc}{(a+b+c)(ab+bc+ca)}\geq 0$

Từ đó ta có $S_{a}+S_{b}\geq 0$. Nếu $S_{a}\geq0$ thì bđt hiển nhiên đúng vì $S_{c}\geq S_{b}\geq S_{a} \geq 0$

Còn nếu $S_{a}\leq 0\Rightarrow S_{b}\geq 0$ (vì $S_{a}+S_{b}\geq 0$)

Ta chú ý tới bđt sau $S_{b}(a-c)^{2}\geq S_{b}(b-c)^{2}\Rightarrow S_{b}(a-c)^{2}+S_{a}(b-c)^{2}\geq (b-c)^{2}(S_{a}+S_{b})\geq 0$

Vậy ta có đpcm

 

Bài toán 14. (Micheal Rozenberg)

Cho $a,b,c$ dương thỏa $a+b+c=1$. Chứng minh

$\frac{a+b}{ab+1}+\frac{b+c}{bc+1}+\frac{c+a}{ca+1}\geq \frac{9}{5}$




#641762 VMF's Marathon Bất Đẳng Thức Olympic

Đã gửi bởi Gachdptrai12 on 22-06-2016 - 19:31 trong Bất đẳng thức và cực trị

Anh đã kiểm tra lại và thấy nó đúng :)

 

201641cf1553-9fff-44c7-9005-1a3f1ae7041b

èo srry em chưa phân tích hết ra  :wacko:  :wacko:




#636326 VMF's Marathon Bất Đẳng Thức Olympic

Đã gửi bởi Gachdptrai12 on 28-05-2016 - 20:11 trong Bất đẳng thức và cực trị

Bài 15. Cho $a,b,c>0$ sao cho $ab+bc+ca=1$. Chứng minh
$$(a^2+ab+b^2)(b^2+bc+c^2)(c^2+ca+a^2)\ge\frac{2}{3}(\frac{a}{b+c}+\frac{b}{c+a}+\frac{c}{a+b}).$$

Lời giải bài 15

Ta sẽ đồng bậc hóa bất đẳng thức

$\Leftrightarrow(a^{2}+ab+b^{2})(b^{2}+bc+c^{2})(c^{2}+ca+a^{2})\ge\frac{2}{3}(\frac{a}{b+c}+\frac{b}{c+a}+\frac{c}{a+b})(ab+bc+ca)^{3}$
Bây giờ ta sẽ chứng minh
$2(\sum \frac{a}{b+c})\leq \frac{2(a^{2}+b^{2}+c^{2})+ab+bc+ca}{ab+bc+ca}$

$\Leftrightarrow 2(\sum \frac{a}{b+c})-3\geq \frac{2(a^{2}+b^{2}+c^{2}-ab-bc-ca)}{ab+bc+ca}$

$\Leftrightarrow \sum \frac{(a-b)^{2}}{(a+c)(b+c)}\leq \frac{\sum(a-b)^{2}}{ab+bc+ca}$

$\Leftrightarrow \sum \frac{c^{2}(a-b)^{2}}{(ab+bc+ca)(a+c)(b+c)}\geq 0$

Vậy bổ đề được chứng minh ta sẽ chứng minh 1 bất đẳng thức mạnh hơn
$(a^{2}+ab+b^{2})(b^{2}+bc+c^{2})(c^{2}+ca+a^{2})\geq \frac{1}{3}(2\sum a^{2}+\sum ab)(ab+bc+ca)^{2}$
$\Leftrightarrow 3(a^{2}+ab+b^{2})(b^{2}+ca+c^{2})(c^{2}+ca+a^{2})-(2\sum a^{2}+\sum ab)(ab+bc+ca)^{2}\geq 0$

$\Leftrightarrow \sum a^{2}b^{2}(a^{2}+b^{2})-abc[\sum a^{3}+\sum ab(a+b)+3abc]+2\sum a^{3}b^{3}\geq 0$
Ta chú ý các bất đẳng thức sau
$2\sum a^{3}b^{3}=\sum a^{3}(b^{3}+c^{3})\geq \sum a^{3}bc(b+c)=\sum abc^{3}(a+b)$
$\sum a^{2}b^{2}(a^{2}+b^{2})=\sum a^{4}(b^{2}+c^{2})\geq 2abc(a^{3}+b^{3}+c^{3})\geq abc(\sum a^{3}+3abc)$
Từ đây ta có điều cần chứng minh

 

Bài toán 16. (Võ Quốc Bá Cẩn,Trần Quang Hùng) Cho $a,b,c$ là các số thực không âm thỏa $a+b+c=1$. Chứng minh

$\dfrac{1}{\sqrt{(a^2+ab+b^2)(a^2+ac+c^2)}}+\dfrac{1}{\sqrt{(b^2+bc+c^2)(b^2+ba+a^2)}}+\dfrac{1}{\sqrt{(c^2+ca+a^2)(c^2+cb+b^2)}}\geq 4+\dfrac{8}{\sqrt{3}}$




#635455 VMF's Marathon Bất Đẳng Thức Olympic

Đã gửi bởi Gachdptrai12 on 25-05-2016 - 15:59 trong Bất đẳng thức và cực trị

 

 Bài toán 7. (VMF) Cho $a,b,c\geq 0$ trong đó không có hai số nào đồng thời bằng 0 và số thực $k$ thoả mãn $3^k\geq 2^{k+1}$. Chứng minh rằng : 

\[\frac{1}{a^k+b^k}+\frac{1}{b^k+c^k}+\frac{1}{c^k+a^k}\geq \frac{5.2^{k-1}}{(a+b+c)^k}\]

 

 Lời giải bài 7: Ta có bổ đề $\frac{2}{x^{k}+y^{k}}+\frac{1}{x^{k}}+\frac{1}{y^{k}}\geq \frac{3.2^{k}}{(x+y)^{k}}$

 Chứng minh áp dụng AM-GM ta có 

$\frac{2}{x^{k}+y^{k}}+\frac{x^{k}+y^{k}}{2x^{k}y^{k}}+\frac{x^{k}+y^{k}}{2x^{k}y^{k}}\geq \frac{2}{x^{k}+y^{k}}+\frac{x^{k}+y^{k}}{2x^{k}y^{k}}+\frac{1}{x^{\frac{k}{2}}y^{\frac{k}{2}}}\geq 3\sqrt[3]{\frac{1}{(xy)\frac{3k}{2}}}\geq \frac{3.2^{k}}{(x+y)^{k}}$

Đúng, vậy bổ đề được chứng minh

Ta có giả thiết được viết lại là $k\geq \log_{\frac{3}{2}}2$ 

Không mất tính tổng quát giả sử $a\geq b\geq c$ ta có các nhận xét

$(a+\frac{c}{2})^{k}\geq a^{k}+c^{k}$

$(b+\frac{c}{2})^{k}\geq b^{k}+c^{k}$

Thật vậy với c=0 thì các nhận xét hiển nhiên đúng với c khác 0 ta có bất đẳng thức trên tương đương 
$(t+\frac{1}{2})^{k}-t^{k}-1\geq 0$ và $t=\frac{a}{c}\geq 1$
Xét hàm $f(t)=(t+\frac{1}{2})^{k}-t^{k}-1$ với $t\in [1;+\infty )\Rightarrow f'(t)=k[(t+\frac{1}{2})^{k-1}-t^{k-1}]>0$
Vậy $f(t)$ đồng biến trên$[1;+\infty )$
Vậy $f(t)\geq f(1)=(\frac{3}{2})^{k}-2\geq (\frac{3}{2})^{\log_{\frac{3}{2}^{2}}}-2=0$ vậy nhận xét đc chứng minh
Vậy $\sum \frac{1}{(a+\frac{c}{2})^{k}}\geq \frac{1}{(a+\frac{c}{2})^{k}}+\frac{1}{(b+\frac{c}{2})^{k}}+\frac{1}{a^{k}+b^{k}}\geq \frac{3.2^{k}}{(a+b+c)^{k}}-\frac{2}{(a+\frac{c}{2})^{k}+(b+\frac{c}{2})^{k}}+\frac{1}{a^{k}+b^{k}}$
Ta cần chứng minh
$\frac{3.2^{k}}{(a+b+c)^{k}}-\frac{2}{(a+\frac{c}{2})^{k}+(b+\frac{c}{2})^{k}}+\frac{1}{a^{k}+b^{k}}\geq \frac{5.2^{k-1}}{(a+b+c)^{k}}$
$\Leftrightarrow \frac{2^{^{k-1}}}{(a+b+c)^{k}}+\frac{1}{a^{k}+b^{k}}\geq \frac{2}{(a+\frac{c}{2})^{k}+(b+\frac{c}{2})^{k}}$
Theo bất đẳng thức holder ta có
$2^{k-1}[(a+\frac{c}{2})^{k}+(b+\frac{c}{2})^{k}]\geq (a+b+c)^{k}$
mà hiển nhiên ta có$(a+\frac{c}{2})^{k}+(b+\frac{c}{2})^{k}\geq (a+b)^{k}$
Vậy ta có đpcm 
 
 \begin{array}{| l | l |} \hline \text{HDTterence2k} & 1\\ \hline \text{hoanglong2k} & 3\\ \hline \text{Gachdptrai12} & 4\\ \hline \text{Nguyenhuyen_AG} & 3\\ \hline \text{fatcat12345} & 1\\ \hline \text{lenhatsinh3} & 1\\ \hline\end{array} 



#636646 VMF's Marathon Bất Đẳng Thức Olympic

Đã gửi bởi Gachdptrai12 on 29-05-2016 - 21:39 trong Bất đẳng thức và cực trị

 Bài toán 17. (Nguyễn Quốc Anh) Cho các số thực dương $x,y,z$ thỏa mãn $x+y+z=32$. Tìm giá trị lớn nhất của

$P=x^3y+y^3z+z^3x$

 Lời giải : Giả sử $x=\max \{x,y,z\}$

Ta có

$a^{3}b+b^{3}c+c^{3}a\leq a^{3}b+a^{2}bc+a^{2}c^{2}\leq a^{3}b+a^{2}bc+\frac{a^{2}c(a+c)}{2}=a^{2}(a+c)\left(b+\frac{c}{2}\right)$

Áp dụng AM-GM ta có

$a^{2}(a+c)\left(b+\frac{c}{2}\right)=27\frac{a^{2}(a+c)}{27}\left(b+\frac{c}{2}\right)\leq \frac{27}{256}\left(\frac{a}{3}+\frac{a}{3}+\frac{a+c}{3}+b+\frac{c}{2}\right)^{4}= \frac{27}{256}\left(a+b+\frac{5c}{6}\right)^{4}\leq \frac{27}{256}(a+b+c)^{4}=\frac{32^{4}.27}{256}$




#638208 VMF's Marathon Bất Đẳng Thức Olympic

Đã gửi bởi Gachdptrai12 on 05-06-2016 - 10:13 trong Bất đẳng thức và cực trị

Bài 25.

Giả sử $a\geq b\geq c,$

$\frac{4(a^2+b^2+c^2)}{ab+bc+ca}+\sqrt{3}\sum_{cyc}\frac{a+2b}{\sqrt{a^2+2b^2}}-13$
$=\sum_{cyc}\left ( \frac{1}{ab+bc+ca}-\frac{3}{(b+2c+\sqrt{3(b^2+2c^2)})\sqrt{3(b^2+2c^2)}} \right )(b-c)^2$
$\geq \left ( \frac{1}{ab+bc+ca}-\frac{3}{(c+2a+\sqrt{3(c^2+2a^2)})\sqrt{3(c^2+2a^2)}} \right )(c-a)^2+\left ( \frac{1}{ab+bc+ca}-\frac{3}{(b+2c+\sqrt{3(b^2+2c^2)})\sqrt{3(b^2+2c^2)}} \right )(b-c)^2$
$\geq \frac{1}{b^2}\left ( \frac{a^2}{ab+bc+ca}-\frac{3a^2}{(c+2a+\sqrt{3(c^2+2a^2)})\sqrt{3(c^2+2a^2)}}+\frac{b^2}{ab+bc+ca}-\frac{3b^2}{(b+2c+\sqrt{3(b^2+2c^2)})\sqrt{3(b^2+2c^2)}} \right )(b-c)^2$
$\geq \frac{1}{b^2}\left ( \frac{a^2+b^2}{ab+bc+ca}-\frac{3a^2}{2(c+2a)^2}-\frac{3b^2}{2(b+2c)^2} \right )(b-c)^2$
$\geq \frac{(8a^4b^2+8a^2b^4-15a^3b^3)+(19a^4c^2-19a^3b^2c)+(4a^4bc-4a^2bc^3)}{2b^2(ab+bc+ca)(c+2a)^2(b+2c)^2}\geq 0\blacksquare$

 Bách coi lại bài này đâu thể giả sử $a\geq b\geq c$ và cái tô đỏ thứ 2 là chưa chắc lớn hơn




#636647 VMF's Marathon Bất Đẳng Thức Olympic

Đã gửi bởi Gachdptrai12 on 29-05-2016 - 21:45 trong Bất đẳng thức và cực trị

P/s. Anh nghĩ lời giải của Long không đúng, bài này chắc có kiểu dồn biến theo kiểu thừa trừ của anh Cẩn. Còn điều kiện của bài 17dương hay không âm nhỉ ?

lời giải của Long em nghĩ nó đúng mà với lại đó là dồn biến thừa trừ luôn mà anh Huyện




#639661 VMF's Marathon Bất Đẳng Thức Olympic

Đã gửi bởi Gachdptrai12 on 11-06-2016 - 20:52 trong Bất đẳng thức và cực trị

 

Lời giải của em là viết tắt lại lời giải ở link sau, Gachdptrai12: http://www.artofprob...h545892p3248382

Xin được giới thiệu lời giải khá hay sau của bạn Phạm Nguyễn Mạnh (với ý tưởng tương tự):

Đặt $X=\sqrt{a^2+b^2+c^2},\ Y=\sqrt{ab+bc+ca}$. Viết bất đẳng thức cần chứng minh dưới dạng

$$\sum \left(X-\sqrt{a^2+2bc}\right) \geq 2(X-Y),$$
$$\sum \dfrac{(b-c)^2}{X+\sqrt{a^2+2bc}} \geq \dfrac{\sum (b-c)^2}{X+Y}.$$
Sử dụng bất đẳng thức Cauchy-Schwarz, ta có
$$\sum \dfrac{(b-c)^2}{X+\sqrt{a^2+2bc}} \geq \dfrac{\left[\sum (b-c)^2\right]^2}{\sum (b-c)^2(X+\sqrt{a^2+2bc})}.$$
Ta chỉ cần chứng minh
$$ \dfrac{\sum (b-c)^2}{\sum (b-c)^2(X+\sqrt{a^2+2bc})} \geq \dfrac{1}{X+Y},$$
$$ \sum (b-c)^2(Y-\sqrt{a^2+2bc})\geq 0.$$
Ta có
$$ \sum\left[(b-c)^2(ab+bc+ca)-(b-c)^2(a^2+2bc)\right]=\sum \big[(b-c)^2(a-b)(c-a)\big]=0.$$
Do đó
$$ \sum (b-c)^2Y^2=\sum \dfrac{(b-c)^2(Y^2+a^2+2bc)}{2} \geq \sum (b-c)^2Y\sqrt{a^2+2bc}.$$
Ta thu được
$$ \sum (b-c)^2(Y-\sqrt{a^2+2bc})\geq 0.$$ 
Đẳng thức xảy ra khi và chỉ khi $a=b$, hoặc $b=c$, hoặc $c=a$.

 

p/s TST có anh nào giải ra ko ạ :3 và bài 29 còn 1 cách bằng pp AM-GM của anh Cẩn 




#637156 VMF's Marathon Bất Đẳng Thức Olympic

Đã gửi bởi Gachdptrai12 on 31-05-2016 - 16:37 trong Bất đẳng thức và cực trị

Lời giải bài 21. Lời giải cuả mình phần đầu giống anh Huyện nên mình bỏ phần đầu nhé
Ta chứng minh
$\sum \frac{5}{2a^2+\sqrt{4a^4+5abc}}\ge 3$
Tuy nhiên áp dụng C-S
${\sum \frac{5}{2a^2+\sqrt{4a^4+5abc}}=\sum \frac{5b^2c^2}{2a^2b^2c^2+b^2c^2\sqrt{4a^4+5abc}}\ge \frac{5(ab+bc+ca)^2}{6a^2b^2c^2+\sum b^2c^2\sqrt{4a^4+5abc}}}$
Ta sẽ chứng minh
$5(\frac{1}{a}+\frac{1}{b}+\frac{1}{c})^2\ge 18+\sum 3\frac{\sqrt{4a^4+5abc}}{a^2}$
Áp dụng AM-GM
$3\frac{\sqrt{4a^4+5abc}}{a^2}\le \frac{9+4a^4+5abc}{2a^2}$
Ta chứng minh$\sum \frac{1}{2a^2}+10\sum \frac{1}{ab}\ge 2\sum a^2+3+\frac{5}{2}\sum \frac{1}{a}\sum ab$
Là 1 bất đẳng thức đúng khi $a+b+c=3$




#640778 VMF's Marathon Bất Đẳng Thức Olympic

Đã gửi bởi Gachdptrai12 on 16-06-2016 - 23:27 trong Bất đẳng thức và cực trị

Ta có
\[2\left(\frac{a}b{+\frac{b}c{+\frac{c}{a}}}\right)-1-\frac{21(a^{2}+b^{2}+c^{2})}{(a+b+c)^{2}} = \frac{2\displaystyle \sum a(a-b)^2(b-2c)^2}{abc(a+b+c)^2} \geqslant 0.\]
Từ đó suy ra điều phải chứng minh.

phân tích hình như bị sai á anh



#637814 VMF's Marathon Bất Đẳng Thức Olympic

Đã gửi bởi Gachdptrai12 on 03-06-2016 - 16:03 trong Bất đẳng thức và cực trị

 Bài toán 23. (Sưu tầm) Cho các số thực không âm $x,y,z$ thỏa mãn $x^2+y^2+z^2+2xyz=1$. Chứng minh rằng

\[x^3y+y^3z+z^3x\leq \dfrac{3\sqrt{3}}{16}\]

ta chỉ cần chứng minh bdt khi $x\geq y\geq z$ 

ta có $x\geq \frac{1}{2}\Rightarrow 1=x^{2}+y^{2}+z^{2}+2xyz\geq x^{2}+y^{2}+z^{2}+yz$

từ đó ta có $x^{3}y+y^{3}z+z^{3}x\leq xy(x^{2}+xy+y^{2})=S$

mà theo bddt AM-GM ta có

$S^{2}=x^{2}y^{2}(x^{2}+zy+z^{2})^{2}=27\frac{x^{2}}{3}y^{2}(\frac{x^{2}+yz+z^{2}}{3})^{2}\leq \frac{27}{256}(\frac{x^{2}}{3}+\frac{2(x^{2}+yz+z^{2})}{3}+y^{2})^{4}= \frac{27}{256}(x^{2}+y^{2}+\frac{2(yz+z^{2})}{3})^{4}\leq \frac{27}{256}(x^{2}+y^{2}+z^{2}+yz)^{4}\leq \frac{27}{256}\Rightarrow S\leq \frac{3\sqrt{3}}{16}$

vậy ta có đpcm

 

Về cơ bản thì bài toán này có thể dử dụng ý tưởng như bài  toán 19, 20. Tất nhiên bài này lời giải khá đơn giản vì chỉ có 1 dấu đẳng thức xảy ra (không tính hoán vị).

em nghĩ nó không dễ đâu ạ dù chỉ có 1 dấu bằng nhưng rất khó đoán phải dùng máy tính :3 :v

em đang suy nghĩ 1 cách khác đó là chứng minh $f(\sqrt{\frac{1-z}{2}},\sqrt{\frac{1-z}{2}},z)\geq \frac{3\sqrt{3}}{16}$




#641636 VMF's Marathon Bất Đẳng Thức Olympic

Đã gửi bởi Gachdptrai12 on 21-06-2016 - 19:37 trong Bất đẳng thức và cực trị

Ta có

\[2\left(\frac{a}b{+\frac{b}c{+\frac{c}{a}}}\right)+1-\frac{21(a^{2}+b^{2}+c^{2})}{(a+b+c)^{2}} = \frac{2\displaystyle \sum a(a-b)^2(b-2c)^2}{abc(a+b+c)^2} \geqslant 0.\]

Từ đó suy ra điều phải chứng minh.

hình như cái phân tích thế nào ấy em phân tích nó ra khác và ko lớn hơn 0 anh xem lại dùm em cảm ơn 




#637923 VMF's Marathon Bất Đẳng Thức Olympic

Đã gửi bởi Gachdptrai12 on 03-06-2016 - 22:25 trong Bất đẳng thức và cực trị

Lời giải này ý tưởng đúng nhưng gõ sai nhiều chỗ, em nên viết nghiêm túc hơn. Ý tưởng lời giải anh nói ở trên là quy về chứng minh bất đẳng thức $1$ biến:

$$ x^3y+y^3z+z^3x \leq y(x^3+z^3+xyz) \leq y(x^2+z^2+2xyz)^{\frac{3}{2}}=y(1-y^2)^{\frac{3}{2}} \leq \frac{3\sqrt{3}}{16}.$$
Phần chứng minh $x^3+z^3+xyz\leq (x^2+z^2+2xyz)^{\frac{3}{2}}$ cơ bản là giống với lời giải của em. Chú ý đây là bất đẳng thức hoán vị nên không dùng dồn biến kiểu quy về chứng minh $f(\sqrt{\frac{1-z}{2}},\sqrt{\frac{1-z}{2}},z)\leq \frac{3\sqrt{3}}{16}$ như em nói được. 

 

chắc có lẽ cách em sai 




#642351 VMF's Marathon Bất Đẳng Thức Olympic

Đã gửi bởi Gachdptrai12 on 26-06-2016 - 22:05 trong Bất đẳng thức và cực trị

Lời giải bài 37.

Giả sử $c=min\left \{ a,b,c \right \}.$

Ta chứng minh

$$\sqrt{a^2+bc+b^2}+\sqrt{c^2+ab+a^2}\geq \sqrt{a^2+ab+b^2}+\sqrt{c^2+bc+a^2}$$

bình phương và thu gọn ta cần chứng minh

$$(a^2+bc+b^2)(c^2+ab+a^2)\geq (a^2+ab+b^2)(c^2+bc+a^2)$$

$$\Leftrightarrow b(b+c)(b-c)(a-c)\geq 0$$

Ta cũng chứng minh

$$\sqrt{b^2+ca+c^2}+\sqrt{a^2+bc+c^2}\geq \sqrt{b^2+bc+c^2}+\sqrt{c^2+ca+a^2}$$

bình phương và thu gọn ta cần chứng minh

$$(b^2+ca+c^2)(a^2+bc+c^2)\geq (b^2+bc+c^2)(c^2+ca+a^2)$$

$$\Leftrightarrow c(a+b)(a-b)^2\geq 0$$

Từ 2 bất đẳng thức trên ta dễ dàng suy ra

$$\sum_{cyc}\sqrt{a^2+bc+b^2}\geq \sum_{cyc}\sqrt{a^2+ab+b^2}$$

nên ta chỉ cần chứng minh

$$\sum_{cyc}\sqrt{a^2+ab+b^2}\geq \sqrt{4(a^2+b^2+c^2)+5(ab+bc+ca)}$$

bình phương và thu gọn ta cần chứng minh

$$\sum_{cyc}\sqrt{(a^2+ab+b^2)(b^2+bc+c^2)}\geq (a+b+c)^2$$

ta lại có $(a^2+ab+b^2)(b^2+bc+c^2)-\left [ b^2+\frac{b(c+a)}{2}+ca \right ]^2=\frac{3}{4}b^2(c-a)^2\geq 0$

nên

$$\sum_{cyc}\sqrt{(a^2+ab+b^2)(b^2+bc+c^2)}\geq \sum_{cyc}\left [ b^2+\frac{b(c+a)}{2}+ca \right ]=(a+b+c)^2$$

Xong!

lời giải của bạn là lời giải được lấy từ http://www.artofprob...h507278p2850887




#638079 VMF's Marathon Bất Đẳng Thức Olympic

Đã gửi bởi Gachdptrai12 on 04-06-2016 - 21:07 trong Bất đẳng thức và cực trị

Vì không thấy ai đăng tiếp, mình xin góp một bài, xin lấy tiêu đề là bài toán 24, nếu các bạn không đồng ý thì có thể bỏ bài toán này!

 

Bài toán 24: (Nguồn: Juliel)

Cho các số thực dương a,b,c. Chứng minh:

$$\sqrt[7]{\frac{a^{7}+b^{7}}{2}}+\sqrt[7]{\frac{b^{7}+c^{7}}{2}}+\sqrt[7]{\frac{c^{7}+a^{7}}{2}}\leq \left ( a+b+c \right )^{10}\left ( \frac{1}{9a}+\frac{1}{9b}+\frac{1}{9c} \right )^{9}$$

 

Áp dụng BĐT AM-GM :

$\dfrac{a^7+b^7}{(a+b)^6}+\dfrac{a+b}{2^6}+\dfrac{a+b}{2^6}+\dfrac{a+b}{2^6}+\dfrac{a+b}{2^6}+\dfrac{a+b}{2^6}+\dfrac{a+b}{2^6}\geq 7\sqrt[7]{\dfrac{a^7+b^7}{(2^6)^6}}=\dfrac{7}{32}\sqrt[7]{\dfrac{a^7+b^7}{2}}$

Tức là :
$\dfrac{a^7+b^7}{(a+b)^6}+\dfrac{3}{32}(a+b)\geq \dfrac{7}{32}\sqrt[7]{\dfrac{a^7+b^7}{2}}$
Ta có :
$(a+b)^7=a^7+b^7+7ab(a^5+b^5)+21a^2b^2(a^3+b^3)+35a^3b^3(a+b)=a^7+b^7+7ab\left [ a^5+b^5+3ab(a^3+b^3)+5a^2b^2(a+b) \right ]$
Theo AM-GM :
$12a^5+9b^5+7a^4b+2ab^4\geq 30\sqrt[30]{(a^5)^{12}.(b^5)^9.(a^4b)^7.(ab^4)^2}=30a^3b$
$12b^5+9a^5+7b^4a+2ba^4\geq 30\sqrt[30]{(b^5)^{12}.(a^5)^9.(b^4a)^7.(ba^4)^2}=30b^3a$
Cộng vế theo vế hai kết quả trên thì được :
$7(a^5+b^5)+3ab(a^3+b^3)\geq 10a^2b^2(a+b)$
Suy ra :
 
$16(a^5+b^5)+48ab(a^3+b^3)+80a^2b^2(a+b)\geq 9(a^5+b^5)+45ab(a^3+b^3)+90a^2b^2(a+b)\Leftrightarrow 16\left [ a^5+b^5+3ab(a^3+b^3)+5a^2b^2(a+b) \right ]\geq 9\left [ a^5+b^5+5ab(a^3+b^3)+10a^2b^2(a+b) \right ]\Leftrightarrow a^5+b^5+3ab(a^3+b^3)+5a^2b^2(a+b) \geq \dfrac{9}{16}(a+b)^5$
Từ đó ta được :
 
$(a+b)^7\geq a^7+b^7+7ab.\dfrac{9}{16}(a+b)^5\Leftrightarrow \dfrac{a^7+b^7}{(a+b)^6}\leq a+b-\dfrac{63ab}{16(a+b)}$
Lại được tiếp :
$\dfrac{7}{32}\sqrt[7]{\dfrac{a^7+b^7}{2}}\leq \dfrac{3}{32}(a+b)+(a+b)-\dfrac{63ab}{16(a+b)}=\dfrac{35}{32}(a+b)-\dfrac{63ab}{16(a+b)}\Leftrightarrow \sqrt[7]{\dfrac{a^7+b^7}{2}}\leq 5(a+b)-\dfrac{18ab}{a+b}$
Thiết lập các BĐT tương tự rồi cộng chúng vế theo vế, ta dẫn đến việc chứng minh :
$\left ( a+b+c \right )^{10}\left ( \dfrac{1}{9a}+\dfrac{1}{9b} +\dfrac{1}{9c}\right )^9+\dfrac{18ab}{a+b}+\dfrac{18bc}{b+c}+\dfrac{18ca}{c+a}\geq 10(a+b+c)\Leftrightarrow \dfrac{(a+b+c)^{10}.(ab+bc+ca)^9}{(abc)^9.9^9}+\dfrac{18}{\dfrac{1}{a}+\dfrac{1}{b}}+\dfrac{18}{\dfrac{1}{b}+\dfrac{1}{c}}+\dfrac{18}{\dfrac{1}{c}+\dfrac{1}{a}}\geq 10(a+b+c)$
Theo Cauchy-Schwarz :
$\dfrac{18}{\dfrac{1}{a}+\dfrac{1}{b}}+\dfrac{18}{\dfrac{1}{b}+\dfrac{1}{c}}+\dfrac{18}{\dfrac{1}{c}+\dfrac{1}{a}}\geq \dfrac{18.9}{2\left ( \dfrac{1}{a}+\dfrac{1}{b} +\dfrac{1}{c}\right )}=\dfrac{81abc}{ab+bc+ca}$
Như vậy cần chỉ ra :
$\dfrac{(a+b+c)^{10}.(ab+bc+ca)^9}{(abc)^9.9^9}+\dfrac{81abc}{ab+bc+ca}\geq 10(a+b+c)$
Nhưng điều này hiển nhiên đúng theo AM-GM cho 10 số :
$\dfrac{(a+b+c)^{10}.(ab+bc+ca)^9}{(abc)^9.9^9}+\dfrac{81abc}{ab+bc+ca}=\dfrac{(a+b+c)^{10}.(ab+bc+ca)^9}{(abc)^9.9^9}+9.\dfrac{9abc}{ab+bc+ca}\geq 10\sqrt[10]{\dfrac{(a+b+c)^{10}.(ab+bc+ca)^9}{(abc)^9.9^9}.\dfrac{(9abc)^9}{(ab+bc+ca)^9}}=10(a+b+c)$
p/s:đề nghị sửa đề  :D  :D



#642374 VMF's Marathon Bất Đẳng Thức Olympic

Đã gửi bởi Gachdptrai12 on 26-06-2016 - 23:59 trong Bất đẳng thức và cực trị

bài 38)cho a,b,c là các số thực ko âm và $a^{2}+b^{2}+c^{2}=3$ chứng minh
$\sum \frac{a^{2}+3b^{2}}{a+3b}\geq 3$